LSAT and Law School Admissions Forum

Get expert LSAT preparation and law school admissions advice from PowerScore Test Preparation.

User avatar
 Dave Killoran
PowerScore Staff
  • PowerScore Staff
  • Posts: 5853
  • Joined: Mar 25, 2011
|
#27190
Complete Question Explanation
(The complete setup for this game can be found here: lsat/viewtopic.php?t=11429)

The correct answer choice is (D)

Since L is scheduled for day 3, P cannot be scheduled for day 2 (because of the contrapositive of the last rule) or day 3 (because of the third rule). Since O is scheduled for day 2, again because of the second rule, P cannot be scheduled for day 1. It follows that answer choice (D) is correct.

Get the most out of your LSAT Prep Plus subscription.

Analyze and track your performance with our Testing and Analytics Package.